Diễn Đàn MathScopeDiễn Đàn MathScope
  Diễn Đàn MathScope
Ghi Danh Hỏi/Ðáp Thành Viên Social Groups Lịch Ðánh Dấu Ðã Ðọc

Go Back   Diễn Đàn MathScope > Sơ Cấp > Lý Thuyết Số > Chuyên Đề

News & Announcements

Ngoài một số quy định đã được nêu trong phần Quy định của Ghi Danh , mọi người tranh thủ bỏ ra 5 phút để đọc thêm một số Quy định sau để khỏi bị treo nick ở MathScope nhé !

* Nội quy MathScope.Org

* Một số quy định chung !

* Quy định về việc viết bài trong diễn đàn MathScope

* Nếu bạn muốn gia nhập đội ngũ BQT thì vui lòng tham gia tại đây

* Những câu hỏi thường gặp

* Về việc viết bài trong Box Đại học và Sau đại học


Trả lời Gởi Ðề Tài Mới
 
Ðiều Chỉnh Xếp Bài
Old 03-01-2008, 06:55 PM   #1
n.t.tuan
+Thành Viên+
 
n.t.tuan's Avatar
 
Tham gia ngày: Nov 2007
Bài gởi: 1,250
Thanks: 119
Thanked 616 Times in 249 Posts
Khi nào một đa thức là bất khả quy?

Topic này sẽ giới thiệu vài tiêu chuẩn bất khả quy của các đa thức , tôi không đăng các chứng minh kèm theo vì hoặc là các bạn đều biết rồi hoặc kiến thức của các bạn học sinh phổ thông không thể hiểu các chứng minh đó. Nếu có ai đó yêu cầu một chứng minh , tôi sẽ đăng nó nếu tôi biết.

k ở đây là Z,Q,... (nhìn như một vành)
Định nghĩa. Một đa thức f với hệ số trong k được gọi là khả quy trên k nếu f=gh, ở đây g và h là các đa thức bậc dương và có hệ số thuộc k. Trong các trường hợp khác f được gọi là bất khả quy trên k.

Giờ tôi đi vào giới thiệu luôn các tiêu chuẩn.

Định lý 1. (Eisenstein)
$f(x)=a_0+a_1x+...+a_nx^n\in\mathbb{Z}[x] $. Nếu có số nguyên tố p sao cho $a_n $ không chia hết cho p , $a_0,...,a_{n-1} $ chia hết cho p, và $a_0 $ không chia hết cho$ p^2 $ thì f là bất khả quy trên Z.
[RIGHT][I][B]Nguồn: MathScope.ORG[/B][/I][/RIGHT]
 
__________________
T.
n.t.tuan is offline   Trả Lời Với Trích Dẫn
The Following 2 Users Say Thank You to n.t.tuan For This Useful Post:
Conanvn (20-01-2013), sonzqnn (17-01-2009)
Old 03-01-2008, 07:25 PM   #2
psquang_pbc
+Thành Viên Danh Dự+
 
psquang_pbc's Avatar
 
Tham gia ngày: Nov 2007
Bài gởi: 747
Thanks: 9
Thanked 111 Times in 72 Posts
Gửi tin nhắn qua Yahoo chát tới psquang_pbc
Chứng minh của Định lý 1 :

Giả sử lại rằng :

$P(x)=Q(x).R(x).(*) $

ở đây $Q(x) $ và $R(x) $ là các đa thức hệ số nguyên và :

$Q(x)=\sum_{i=0}^hb_ix^i $

$R(x)=\sum_{i=0}^kc_ix^i $

với $b_i\;(i=\overline{0,h})\;,c_j\;(j=\overline{0,k})\ $ là các số nguyên, $h,k>0,h+k=n $

Bằng cách đồng nhất hệ số của 2 vế đẳng thức $(*) $ ta có $a_0=b_h.c_k $. Do $a_0\;\vdots\;p $ nhưng $a\;\not\vdots\;p^2 $ nên hoặc $b_h\;\vdots \;p $ hoặc $c_k\;\vdots \;p. $

Giả sử $b_h\;\vdots \;p $.Ta có chuỗi các đẳng thức sau:

$a_{1}=b_hc_{k-1}+b_{h-1}c_k $

$a_{2}=b_hc_{k-2}+b_{h-1}c_{k-1}+b_{h-2}c_k $

$a_{h}=a_{n-k}=b_{h}.c_{k-h}+b_{h-1}.c_{k-h+1}+...+b_1.c_{k-1}+b_0.c_k $

Trong đẳng thức thứ nhất ta suy ra $b_{h-1}\;\vdots\;p $, tương tự ta cũng có $b_{h-2}\;\vdots\;p $ từ đẳng thức thứ 2. Quá trình đó tiếp tục đến lúc ta có $b_0\;\vdots\;p $ hay $a_n\;\vdots\;p $ vô lí.

Kết thúc chứng minh.

[RIGHT][I][B]Nguồn: MathScope.ORG[/B][/I][/RIGHT]
 
__________________
[Only registered and activated users can see links. ]

No pain, no gain!

thay đổi nội dung bởi: psquang_pbc, 04-01-2008 lúc 06:15 AM
psquang_pbc is offline   Trả Lời Với Trích Dẫn
The Following 3 Users Say Thank You to psquang_pbc For This Useful Post:
Conanvn (20-01-2013), duonglangquyen (09-12-2010), sonzqnn (17-01-2009)
Old 03-01-2008, 09:47 PM   #3
n.t.tuan
+Thành Viên+
 
n.t.tuan's Avatar
 
Tham gia ngày: Nov 2007
Bài gởi: 1,250
Thanks: 119
Thanked 616 Times in 249 Posts
Định lý 2.
Cho p là một số nguyên tố. Nếu đa thức $f(x)=a_nx^n+...+a_0\in\mathbb{Z}[x] $ khả quy trên Z, và p chia hết các hệ số $a_0,...,a_{n-2} $ nhưng không chia hết $a_n $, và $p^2 $ không chia hết $a_0 $, thì p không chia hết $a_{n-1} $ và f phải có ít nhất một nghiệm hữu tỷ.
[RIGHT][I][B]Nguồn: MathScope.ORG[/B][/I][/RIGHT]
 
__________________
T.
n.t.tuan is offline   Trả Lời Với Trích Dẫn
Old 04-01-2008, 06:24 AM   #4
psquang_pbc
+Thành Viên Danh Dự+
 
psquang_pbc's Avatar
 
Tham gia ngày: Nov 2007
Bài gởi: 747
Thanks: 9
Thanked 111 Times in 72 Posts
Gửi tin nhắn qua Yahoo chát tới psquang_pbc
Định lý $2 $ là hệ quả của định lý sau :

Định lý $2' $

Cho $P(x)=\sum_{i=0}^n a_{n-i}i.x^i $ là đa thức hệ số nguyên và $k $ là $1 $ số tự nhiên sao cho $0\le k\le n-1 $. Giả sử tồn tại số nguyên tố $p $ sao cho:

$1, $ $a_0 $ không chia hết cho $p $.

$2, $ Những hệ số $a_{k+1},a_{k+2},...,a_n $ chia hết cho $p $.

$3, $ $a_n $ không chia hết cho $p^2 $.

Chứng minh khi đó P(x) có ước không phân tích được là G(x) mà bậc của đa thức này lớn hơn hoặc bằng $n-k. $

Với $k=0 $ ta nhận được tiêu chuẩn Eisenstein

Với $k=1 $ ta có định lý 2 của anh Tuân.

Với $k=n-2 $ thì tồn tại 1 nghiệm của P(x) không là số vô tỉ .

[RIGHT][I][B]Nguồn: MathScope.ORG[/B][/I][/RIGHT]
 
__________________
[Only registered and activated users can see links. ]

No pain, no gain!
psquang_pbc is offline   Trả Lời Với Trích Dẫn
The Following User Says Thank You to psquang_pbc For This Useful Post:
Conanvn (20-01-2013)
Old 04-01-2008, 11:09 AM   #5
T.Courtin
Guest
 
Tham gia ngày: Jan 2008
Bài gởi: 49
Thanks: 1
Thanked 6 Times in 4 Posts
Định lý 3.(Tiêu chuẩn Polya)
Cho $f\in\mathbb{Z}[x] $ bậc $n $ . Đặt $m=\left[\frac{n+1}{2}\right] $ .Giả sử $n $ số nguyên khác nhau $d_1,d_2,\ldots,d_n $ không là nghiệm của $f $ và thỏa mãn $f(d_i)<\frac{m!}{2^n} $ .
Khi đó $f $ bất khả quy.
[RIGHT][I][B]Nguồn: MathScope.ORG[/B][/I][/RIGHT]
 
T.Courtin is offline   Trả Lời Với Trích Dẫn
The Following 2 Users Say Thank You to T.Courtin For This Useful Post:
nguyenhtctb (29-06-2011), nguyentrai_oly (07-04-2012)
Old 04-01-2008, 12:35 PM   #6
psquang_pbc
+Thành Viên Danh Dự+
 
psquang_pbc's Avatar
 
Tham gia ngày: Nov 2007
Bài gởi: 747
Thanks: 9
Thanked 111 Times in 72 Posts
Gửi tin nhắn qua Yahoo chát tới psquang_pbc
Định lý 4.(Tiêu chuẩn Oskar Perron)

Cho đa thức $P(x) $ hệ số nguyên. Giả sử tồn tại số nguyên $b $ và số nguyên tố $p $ sao cho chúng thoả mãn điều kiện sau :

$1, P(b)=p $

$2, P(b-1)\not=0. $

3, Tất cả các nghiệm $\xi_i;\; ( i=\overline{1,n} ) $ của đa thức $P(x) $ thỏa mãn bất đẳng thức $Re\xi_i <b-\frac{1}{2} $. Khi đó đa thức $P(x) $ bất khả quy.

[RIGHT][I][B]Nguồn: MathScope.ORG[/B][/I][/RIGHT]
 
__________________
[Only registered and activated users can see links. ]

No pain, no gain!
psquang_pbc is offline   Trả Lời Với Trích Dẫn
The Following User Says Thank You to psquang_pbc For This Useful Post:
sonzqnn (17-01-2009)
Old 04-01-2008, 06:13 PM   #7
n.t.tuan
+Thành Viên+
 
n.t.tuan's Avatar
 
Tham gia ngày: Nov 2007
Bài gởi: 1,250
Thanks: 119
Thanked 616 Times in 249 Posts
Trích:
Nguyên văn bởi n.t.tuan View Post
Định lý 2.
Cho p là một số nguyên tố. Nếu đa thức $f(x)=a_nx^n+...+a_0\in\mathbb{Z}[x] $ khả quy trên Z, và p chia hết các hệ số $a_0,...,a_{n-2} $ nhưng không chia hết $a_n $, và $p^2 $ không chia hết $a_0 $, thì p không chia hết $a_{n-1} $ và f phải có ít nhất một nghiệm hữu tỷ.
Dùng định lý này có thể giải được các bài 10,11 ở đây //http://ant.edu.ms/forum/showthread.php?t=1490 . Chú Quang post chứng minh của định lý 2 đi.
[RIGHT][I][B]Nguồn: MathScope.ORG[/B][/I][/RIGHT]
 
__________________
T.
n.t.tuan is offline   Trả Lời Với Trích Dẫn
Old 04-01-2008, 06:52 PM   #8
psquang_pbc
+Thành Viên Danh Dự+
 
psquang_pbc's Avatar
 
Tham gia ngày: Nov 2007
Bài gởi: 747
Thanks: 9
Thanked 111 Times in 72 Posts
Gửi tin nhắn qua Yahoo chát tới psquang_pbc
Để thư thả rồi em post chứng minh Định lý 2' luôn. Giờ mệt quá chả làm được gì, lên mạng spam tí thôi
[RIGHT][I][B]Nguồn: MathScope.ORG[/B][/I][/RIGHT]
 
__________________
[Only registered and activated users can see links. ]

No pain, no gain!
psquang_pbc is offline   Trả Lời Với Trích Dẫn
Old 04-01-2008, 07:03 PM   #9
n.t.tuan
+Thành Viên+
 
n.t.tuan's Avatar
 
Tham gia ngày: Nov 2007
Bài gởi: 1,250
Thanks: 119
Thanked 616 Times in 249 Posts
Tùy chú, anh post tiêu chuẩn khác nhé!

Định lý 5.
Cho $f(x)=x^n+a_1x^{n-1}+...+a_n\in\mathbb{Z}[x],a_n\not = 0 $.
a)Nếu $|a_1|>1+|a_2|+...+|a_n| $ thì f là bất khả quy trên Z.
b)Nếu $|a_1|\geq1+|a_2|+...+|a_n| $ và $f(\pm 1)\not = 0 $ thì f là bất khả quy trên Z.
[RIGHT][I][B]Nguồn: MathScope.ORG[/B][/I][/RIGHT]
 
__________________
T.
n.t.tuan is offline   Trả Lời Với Trích Dẫn
The Following 3 Users Say Thank You to n.t.tuan For This Useful Post:
hoangkhtn2010 (04-03-2011), nguyenhtctb (29-06-2011), sonzqnn (17-01-2009)
Old 04-01-2008, 07:10 PM   #10
psquang_pbc
+Thành Viên Danh Dự+
 
psquang_pbc's Avatar
 
Tham gia ngày: Nov 2007
Bài gởi: 747
Thanks: 9
Thanked 111 Times in 72 Posts
Gửi tin nhắn qua Yahoo chát tới psquang_pbc
Tiếp theo là định lý 6 nhé :

Định lý 6 :

Cho $b\ge 3 $ là 1 số nguyên dương và $p $ là 1 số nguyên tố . Viết $p $ dưới dạng cơ số $b $ như sau :

$p=\sum_{i=0}^na_ib^{n-i} $

ở đây n là số tự nhiên $a_0\not=0 $ và $0\le a_i<b\; ( i=\overline{1,n} \; ) $ . Khi đó đa thức:

$P(x)=\sum_{i=0}^na_ix^{n-i} $

là đa thức bất khả quy.

[RIGHT][I][B]Nguồn: MathScope.ORG[/B][/I][/RIGHT]
 
__________________
[Only registered and activated users can see links. ]

No pain, no gain!
psquang_pbc is offline   Trả Lời Với Trích Dẫn
The Following User Says Thank You to psquang_pbc For This Useful Post:
sonzqnn (17-01-2009)
Old 04-01-2008, 07:18 PM   #11
Lam_sptn
+Thành Viên+
 
Tham gia ngày: Jan 2008
Bài gởi: 3
Thanks: 0
Thanked 0 Times in 0 Posts
Trích:
Nguyên văn bởi psquang_pbc View Post
Định lý $2 $ là hệ quả của định lý sau :

Định lý $2' $

Cho $P(x)=\sum_{i=0}^n a_{n-i}i.x^i $ là đa thức hệ số nguyên và $k $ là $1 $ số tự nhiên sao cho $0\le k\le n-1 $. Giả sử tồn tại số nguyên tố $p $ sao cho:

$1, $ $a_0 $ không chia hết cho $p $.

$2, $ Những hệ số $a_{k+1},a_{k+2},...,a_n $ chia hết cho $p $.

$3, $ $a_n $ không chia hết cho $p^2 $.

Chứng minh khi đó P(x) có ước không phân tích được là G(x) mà bậc của đa thức này lớn hơn hoặc bằng $n-k. $

Với $k=0 $ ta nhận được tiêu chuẩn Eisenstein

Với $k=1 $ ta có định lý 2 của anh Tuân.

Với $k=n-2 $ thì tồn tại 1 nghiệm của P(x) không là số vô tỉ .
Ai có thể post giúp chứng minh của 2' được không?
[RIGHT][I][B]Nguồn: MathScope.ORG[/B][/I][/RIGHT]
 
Lam_sptn is offline   Trả Lời Với Trích Dẫn
Old 04-01-2008, 07:42 PM   #12
n.t.tuan
+Thành Viên+
 
n.t.tuan's Avatar
 
Tham gia ngày: Nov 2007
Bài gởi: 1,250
Thanks: 119
Thanked 616 Times in 249 Posts
Trích:
Nguyên văn bởi n.t.tuan View Post

Định lý 5.
Cho $f(x)=x^n+a_1x^{n-1}+...+a_n\in\mathbb{Z}[x],a_n\not = 0 $.
a)Nếu $|a_1|>1+|a_2|+...+|a_n| $ thì f là bất khả quy trên Z.
b)Nếu $|a_1|\geq1+|a_2|+...+|a_n| $ và $f(\pm 1)\not = 0 $ thì f là bất khả quy trên Z.
Dùng Định lý này có thể giải được bài 14 ở topic ''Bài tập về đa thức bất khả quy'' trong box này.
[RIGHT][I][B]Nguồn: MathScope.ORG[/B][/I][/RIGHT]
 
__________________
T.
n.t.tuan is offline   Trả Lời Với Trích Dẫn
Old 13-03-2009, 05:12 PM   #13
zinxinh
+Thành Viên+
 
zinxinh's Avatar
 
Tham gia ngày: Jan 2009
Bài gởi: 214
Thanks: 65
Thanked 70 Times in 45 Posts
Xét trên trường $Z_{p} $
[RIGHT][I][B]Nguồn: MathScope.ORG[/B][/I][/RIGHT]
 
zinxinh is offline   Trả Lời Với Trích Dẫn
Old 14-03-2009, 10:51 AM   #14
namdung
Administrator

 
Tham gia ngày: Feb 2009
Đến từ: Tp Hồ Chí Minh
Bài gởi: 1,343
Thanks: 209
Thanked 4,066 Times in 778 Posts
Gửi tin nhắn qua Yahoo chát tới namdung
Đề chọn đội tuyển của Trung Quốc vừa rồi có bài: Cho m > n > 1 là các số nguyên lẻ, chứng minh rằng đa thức $x^m + x^n + x + 1 $ bất khả quy.
==============
Trích:
Nguyên văn bởi Lam_sptn View Post
Ai có thể post giúp chứng minh của 2' được không?
Chứng minh của 2' hoàn toàn giống như chứng minh của tiêu chuẩn Eisentein thôi. Bạn cứ theo dõi kỹ là chứng minh được.
[RIGHT][I][B]Nguồn: MathScope.ORG[/B][/I][/RIGHT]
 

thay đổi nội dung bởi: namdung, 14-03-2009 lúc 10:54 AM Lý do: Tự động gộp bài
namdung is offline   Trả Lời Với Trích Dẫn
Old 13-04-2009, 11:07 AM   #15
pte.alpha
+Thành Viên+
 
Tham gia ngày: Apr 2009
Bài gởi: 216
Thanks: 8
Thanked 208 Times in 62 Posts
Tôi mới đọc trên Crux có định lý khá hay sau về điều kiện khả quy của đa thức $F(x^2) $.

Định lý: Đa thức $F(x^2) $ là khả quy trên $Z[x] $ khi và chỉ khi hoặc $F(x) $ khả quy, hoặc $aF(x) = G^2(x) - xH^2(x) $ với $G(x), H(x) $ thuộc $Z[x] $, trong đó a = 1 hoặc -1.

Định lý trên được phát biểu cho vành cơ sở K bất kỳ nhưng tôi phát biểu cho Z cho nó gọn. Trong trường hợp tổng quát, a được thay bằng 1 phần tử đơn vị của K.

Ứng dụng. Chứng minh rằng nếu f(x) thuộc Z[x] là đa thức bất khả quy và |f(0)| không chính phương thì f(x^2) bất khả quy.
[RIGHT][I][B]Nguồn: MathScope.ORG[/B][/I][/RIGHT]
 
pte.alpha is offline   Trả Lời Với Trích Dẫn
The Following 2 Users Say Thank You to pte.alpha For This Useful Post:
Conanvn (20-01-2013), nguyenhtctb (29-06-2011)
Trả lời Gởi Ðề Tài Mới

Bookmarks

Ðiều Chỉnh
Xếp Bài

Quuyền Hạn Của Bạn
You may not post new threads
You may not post replies
You may not post attachments
You may not edit your posts

BB code is Mở
Smilies đang Mở
[IMG] đang Mở
HTML đang Tắt

Chuyển đến


Múi giờ GMT. Hiện tại là 01:27 PM.


Powered by: vBulletin Copyright ©2000-2024, Jelsoft Enterprises Ltd.
Inactive Reminders By mathscope.org
[page compression: 102.42 k/118.41 k (13.50%)]